Zusammenhang Markov/Feller-Prozesse

Neue Frage »

fnsr21 Auf diesen Beitrag antworten »
Zusammenhang Markov/Feller-Prozesse
Hallo! Ich habe folgende Frage

Gegeben haben wir einen Markov-Prozess , der beispielsweise die Bewegung eines Teilchen für schreibt, dass zur Zeit in der Position startet. Für bewegt sich das Teilchen mit konstanter Geschwindigkeit nach links; für nach rechts. Ist , so bewegt sich das Teilchen mit Wahrscheinlichkeit in eine der beiden Richtungen. Formal können wir das so schreiben:





Nun ist zu zeigen, dieser Prozess ist Markov, aber nicht Feller.

Erst zur Definition. Dazu definieren wir für einen MP einen Übergangsoperator



wo die Übergangsfunktion des MP bezeichnet. Gilt für diese Operatoren-Halbgruppe (stetig und beschränkt), dann nennen wir den Prozess Feller-Prozess.

Nun werden wir ja sehen, dass das Ergebnis nicht mehr stetig ist. Intuitiv wird es natürlich in der Form



aussehen und damit hat offensichtlich eine Unstetigkeit in .

Nun meine Frage: Wie kann man es explizit berechnen?

Der Ansatz wäre einfach ausrechnen:



und gleich die Integrationsgebiete aufzuspalten:



Nur stimmt der letzte Term dann offensichtlich nicht...

Mit der Markov-Eigenschaft () komme ich auch nicht so richtig weiter.

MfG.
Zündholz Auf diesen Beitrag antworten »
RE: Zusammenhang Markov/Feller-Prozesse
Hallo,
Ich denke dass du das Integral "falsch" aufteilst.
Ich würde es mit


probieren, dabei ist P(Rest) = 0, nach angabe. Und damit hast du dann denke ich alles was du brauchst um das rauszubekommen was du dir schon gedacht hast.

Hast du obiges selbst aufgestellt, oder ist das eine Aufgabe?
fnsr21 Auf diesen Beitrag antworten »
RE: Zusammenhang Markov/Feller-Prozesse
Zitat:
Original von Zündholz
dabei ist P(Rest) = 0, nach angabe.


Ok, das klappt dann wohl. Noch eine andere Idee, gilt für meine Übergangsfunktion nicht nach Definition



wo das Dirac-Maß? Richtig formal begründen kann ichs noch nicht, aber rein intuitiv gesehen. Damit könnte ich es "sauberer" ausrechnen.

Zitat:
Hast du obiges selbst aufgestellt, oder ist das eine Aufgabe?


Die Formalisierung war gegeben, zeigen muss ich aber, dass die der Übergangsoperator eine Unstetigkeit in der 0 hat. Entnommen ist die Aufgabe aus dem Stoyanov-Buch zu "Counterexamples in probablity".
fnsr21 Auf diesen Beitrag antworten »

Mir erscheint die 2. Methode doch einfacher zu rechnen, und auch klarer. Zumal sofort hervor geht, dass der Prozess dann Markovsch ist.

Vielen Dank für die Hilfe!
Neue Frage »
Antworten »



Verwandte Themen

Die Beliebtesten »
Die Größten »
Die Neuesten »